Le site "intégralsbot"

13

Réponses

  • Quentino37: En réalité, ce n'est une preuve de rien :-D. C'est au mieux, un argument heuristique.
    Tout le monde* se gargarise en parlant de cette "preuve" d'Euler.
    En réalité, Euler a publié une vraie preuve (toujours valable aujourd'hui) de cette égalité magnifique qui n'a rien à voir avec l'argument que tu reprends.

    *: les gens qui ont un peu de culture dans le domaine tout du moins.

    PS:
    On en a déjà parlé ici.
    Euler a été confronté au fait qu'il est difficile d'obtenir des décimales du nombre $\zeta(2)$ en utilisant la formule $\zeta(2)=1+\frac{1}{2^2}+\frac{1}{3^2}+\frac{1}{4^2}+... $ car cette série converge désespérément lentement.
    Il a commencé par trouver une autre expression qui permet d'obtenir sans trop de calculs (Euler n'avait que sa tête, une plume et ce qui ressemblait à du papier pour faire ses calculs) des décimales de ce nombre.
    Quand il a obtenu un certain nombre de décimales, il a reconnu ce nombre*. Il savait donc à ce moment-là empiriquement la valeur de cette série. Il lui restait à trouver une preuve.

    * Il calcule quelques décimales et il se dit ce nombre ressemble à cet autre nombre. Il calcule plus de décimales de $\zeta(2)$ et il vérifie que les nouvelles décimales obtenues coïncident avec celles du nombre candidat. Si cela coïncide sur un certain nombre de décimales tu as bon espoir d'avoir trouvé la bonne valeur de la série mais il reste à trouver une preuve. Généralement quand on sait ce qu'on doit trouver cela aide.
  • Euler devait être ultra fort pour reconnaître le nombre avec sa tête, une plume et ce qui ressemblait à du papier pour faire ses calculs ;) il est vraiment super fort
    Je suis donc je pense 
  • Ce type continuait à faire des mathématiques alors qu'il était aveugle*. Il devait avoir une mémoire phénoménale.
    * il se faisait aider pour écrire.
  • oui :)
    Je suis donc je pense 
  • Si on pose pour tout $\displaystyle 0\leq x\leq 1, F(x)=-\int_0^x \dfrac{\ln(1-t)}{t}dt$.
    On peut montrer que pour tout $x\in [0;1]$: $\displaystyle F(x)=x+\dfrac{x^2}{2^2}+\dfrac{x^3}{3^2}+\dfrac{x^4}{4^2}+...$

    On a donc \begin{align}\zeta(2)&=F(1)\\
    &=-\int_0^1 \dfrac{\ln(1-t)}{t}dt\\
    &=-\int_0^{\frac{1}{2}} \dfrac{\ln(1-t)}{t}dt-\underbrace{\int_{\frac{1}{2}}^1 \dfrac{\ln(1-t)}{t}dt}_{y=1-x}\\
    &=F\left(\frac{1}{2}\right)-\int_0^{\frac{1}{2}} \frac{\ln t}{1-t}dt\\
    &\overset{\text{IPP}}=F\left(\frac{1}{2}\right)+\Big[\ln t\ln(1-t)\Big]_0^{\frac{1}{2}}-\int_0^{\frac{1}{2}}\frac{\ln(1-t)}{t}dt\\
    &=\ln^2 2+2F\left(\frac{1}{2}\right)
    \end{align}

    Or, $\displaystyle F\left(\frac{1}{2}\right)=1+\dfrac{1}{2^2\times 2^2}+\dfrac{1}{2^3\times 3^2}+\dfrac{1}{2^4\times 4^2}+...$
    est une série qui converge rapidement.

    NB: La fonction $F$ est connue aujourd'hui sous le nom de dilogarithme on la note $\text{Li}_2$.
  • On peut faire mieux.

    \begin{align}\zeta(2)&=F(1)\\
    &=-\int_0^1 \frac{\ln(1-t)}{t}dt\\
    &=-\int_0^{\frac{1}{3}} \frac{\ln(1-t)}{t}dt-\underbrace{\int_\frac{1}{3}^1 \frac{\ln(1-t)}{t}dt}_{u=\frac{1-t}{1+t}}\\
    &=F\left(\frac{1}{3}\right)-2\int_0^{\frac{1}{2}} \frac{\ln\left(\frac{2u}{1+u}\right)}{1-u^2}du\\
    &=F\left(\frac{1}{3}\right)-2\underbrace{\int_0^{\frac{1}{2}} \frac{\ln\left(\frac{2}{1+u}\right)}{1-u^2}du}_{x=\frac{1-u}{1+u}}-2\underbrace{\int_0^{\frac{1}{2}} \frac{\ln u}{1-u^2}du}_{\text{IPP}}\\
    &=F\left(\frac{1}{3}\right)-\int_{\frac{1}{3}}^1\frac{\ln(1+x)}{x}dx+\left[\ln\left(\frac{1-u}{1+u}\right)\ln u\right]_0^{\frac{1}{2}}-\int_0^{\frac{1}{2}}\frac{\ln\left(\frac{1-u}{1+u}\right)}{u}du\\
    &=F\left(\frac{1}{3}\right)+F\left(\frac{1}{2}\right)+\ln 2\ln 3+\int_0^{\frac{1}{3}}\frac{\ln(1+u)}{u}du+\int_0^{\frac{1}{2}}\frac{\ln(1+u)}{u}du-\int_0^1\frac{\ln(1+u)}{u}du\\
    \end{align}
    Or, pour tout $x\in [0;1]$,
    \begin{align}\int_0^x\frac{\ln(1+t)}{t}dt&=\underbrace{\int_0^x\frac{\ln(1-t^2)}{t}dt}_{u=t^2}-\int_0^x\frac{\ln(1-t)}{t}dt\\
    &=\frac{1}{2}\int_0^{x^2}\frac{\ln(1-u)}{u}du+F(x)\\
    &=F(x)-\frac{1}{2}F(x^2)
    \end{align}
    Donc:
    \begin{align}\zeta(2)&=\ln 2\ln 3+2F\left(\frac{1}{3}\right)+2F\left(\frac{1}{2}\right)-\frac{1}{2}F\left(\frac{1}{9}\right)-\frac{1}{2}F\left(\frac{1}{4}\right)-\frac{1}{2}\zeta(2)\end{align}
    On a vu plus haut que:
    \begin{align}F\left(\frac{1}{2}\right)&=\frac{1}{2}\left(\zeta(2)-\ln^2 2\right)\end{align}
    Donc:
    \begin{align}\zeta(2)&=\ln 2\ln 3-\ln^2 2+2F\left(\frac{1}{3}\right)-\frac{1}{2}F\left(\frac{1}{9}\right)-\frac{1}{2}F\left(\frac{1}{4}\right)+\frac{1}{2}\zeta(2)\\
    &=\boxed{2\ln 2\ln 3-2\ln^2 2+4F\left(\frac{1}{3}\right)-F\left(\frac{1}{9}\right)-F\left(\frac{1}{4}\right)}
    \end{align}

    NB: les arguments de $F$ ci-dessus sont plus petits que $1/2$ les séries correspondantes convergent encore plus vite.
  • \begin{align}K&=\int_0^\infty \int_0^\infty \dfrac{\ln(xy)}{(1+x^3)(1+y^3)}dxdy\\
    &\overset{u(x)=xy}=\int_0^\infty \int_0^\infty \frac{y^2\ln u}{(y^3+u^3)(1+y^3)}dudy\\
    &=-\int_0^\infty \left[\frac{\ln\left(\frac{u^3+y^3}{1+y^3}\right)}{3(1-u^3)}\right]_0^\infty \ln u \,du\\
    &=-\int_0^\infty \frac{\ln^2 u}{1-u^3}du\\
    \end{align}
    Or, par ailleurs,
    \begin{align}K&\overset{\text{Fubini}}=2\left(\int_0^\infty \frac{1}{1+x^3}dx\right)\left(\int_0^\infty \frac{\ln x}{1+x^3}dx\right)\\
    &=\frac{4}{3\sqrt{3}}\int_0^\infty \frac{\ln x}{1+x^3}dx\\
    \int_0^\infty \frac{\ln^2 x}{x^3-1}dx&=\frac{4}{3\sqrt{3}}\int_0^\infty \frac{\ln x}{1+x^3}dx
    \end{align}
  • \begin{align}\int_0^{\infty} \frac{\ln^2 x}{1+x^3}dx&=\int_0^{1} \frac{\ln^2 x}{1+x^3}dx+\underbrace{\int_1^{\infty} \frac{\ln^2 x}{1+x^3}dx}_{y=\frac{1}{x}}\\
    &=\int_0^{1} \frac{(1+x)\ln^2 x}{1+x^3}dx\\
    &=\int_0^{1} \frac{\ln^2 x}{1-x+x^2}dx\\
    \end{align}
    Et, après, on lit ça.

    NB: en réalité, c'est l'égalité $\displaystyle \int_0^{\infty} \frac{\ln^2 x}{1+x^3}dx=\frac{1}{2}\int_0^{\infty} \frac{\ln^2 x}{1-x+x^2}dx$ qui est utile ici.

    PS:
    J'avais complètement oublié que j'avais calculé l'intégrale $\displaystyle \int_0^{\infty} \frac{\ln^2 x}{1-x+x^2}dx$.
    Je me suis souvenu que pour ce type d'intégrales qu'une technique de calcul pouvait fonctionner.
  • Bonsoir, je ne peux pas résister à l'envie de donner ce calcul de $I = \displaystyle \int_0^{+\infty} \dfrac{\ln (x)^2}{1+x^3} \mathrm dx $:
    En suivant Cartan: Théorie Elémentaire des Fonctions Analytiques ... chez Hermann: si $D= \mathbb{C} -\mathbb{R}^+$:
    $$
    \int_0^{+\infty} \dfrac{\ln (x)^3}{1+x^3} \mathrm dx - \int_0^{+\infty} \dfrac{\ln (x+ 2\pi i)^3}{1+x^3} \mathrm dx \; = \; 2\pi i \sum_{a\in D} \bigl\{ \mathrm {res}_{z=a} \left( \dfrac{\ln (z)^3}{1+z^3}\right) \bigr\} .

    $$ Ceci se prouve en intégrant $\dfrac{\ln (z)^3}{1+z^3}$ sur un contour en forme de trou de serrure.
    On développe le cube :
    $$

    -6\pi i \int_0^{+\infty} \dfrac{\ln (x)^2}{1+x^3} \mathrm dx +12\pi^2 \int_0^{+\infty} \dfrac{\ln (x)}{1+x^3} \mathrm dx +8 \pi^3 i \int_0^{+\infty} \dfrac{1}{1+x^3} \mathrm dx \; = \; 2\pi i \sum_{a\in D} \bigl\{ \mathrm {res} _{z=a} \left( \dfrac{\ln (z)^3}{1+z^3}\right) \bigr\} .

    $$ On sait que (en fait, on applique la même méthode avec le carré du logarithme au lieu du cube...):
    1) $\displaystyle \int_0^{+\infty} \dfrac{\ln (x)}{1+x^3} \mathrm dx = -2\pi ^2/27$.
    2) $\displaystyle \int_0^{+\infty} \dfrac{1}{1+x^3} \mathrm dx = 2\pi \sqrt{3}/9$.
    On sépare partie réelle et imaginaire après avoir divisé par $-6\pi i$:
    $$

    \int_0^{+\infty} \dfrac{\ln (x)^2}{1+x^3} \mathrm dx -\frac{4 \pi^2}{3} \int_0^{+\infty} \dfrac{1}{1+x^3} \mathrm dx \; = \; -\frac{1}{3}\Re \bigl\{ \sum_{a\in D} \mathrm {res} _{z=a} \left( \dfrac{\ln (z)^3}{1+z^3}\right) \bigr\} .



    $$ Il reste à évaluer les résidus correspondant aux pôles $-1$, $\rho = e^{i\pi /3}$ et $\overline{\rho}$ :
    Si $g(z) = \dfrac{1}{1+z^3}$, les résidus de $g$ sont:
    1) $ \mathrm {res}_{z=-1} g(z) = \dfrac{1}{3}$ donc $\mathrm {res}_{z=-1} \left( \ln (z)^3 g(z)\right) = \dfrac{(i\pi)^3}{3} = \dfrac{-i\pi^3}{3} $
    2) $ \mathrm {res}_{z=\rho} g(z) = -\dfrac{\rho}{3}$ donc $\mathrm {res}_{z=\rho} \left( \ln (z)^3 g(z)\right) = -\rho\dfrac{(i\pi/3)^3}{3} = i\rho \dfrac{\pi^3}{81}$ donc $\Re \left( i\rho \dfrac{\pi^3}{81}\right) = - \dfrac{\pi^3 \sqrt{3}}{2\times 81}$
    3) $\displaystyle \mathrm {res}_{z=\overline{\rho} } \; g(z) = -\dfrac{\overline{\rho}}{3}$ donc $\mathrm {res}_{z=\overline{\rho}} \left( \ln (z)^3 g(z)\right) = -\overline{\rho}\; \dfrac{(5i\pi/3)^3}{3} = i \; \overline{\rho} \; \dfrac{125\pi^3}{81}$ donc $\Re \left( i\overline{\rho} \dfrac{125\pi^3}{81}\right) = \dfrac{125\pi^3 \sqrt{3}}{2\times 81}$

    On termine le calcul :
    $$ \begin{align*}
    \int_0^{+\infty} \dfrac{\ln (x)^2}{1+x^3} \mathrm dx &= \frac{4 \pi^2}{3} \int_0^{+\infty} \dfrac{1}{1+x^3} \mathrm dx \; -\; \dfrac{62\pi^3 \sqrt{3}}{243} \\


    % \int_0^{+\infty} \dfrac{\ln (x)^2}{1+x^3} \mathrm dx
    &= \frac{4 \pi^2}{3} \times 2\pi \sqrt{3}/9 \; - \; \dfrac{62\pi^3 \sqrt{3}}{243}

    \end{align*}
    $$ Finalement :
    $$
    \int_0^{+\infty} \dfrac{\ln (x)^2}{1+x^3} \mathrm dx \; = \; \dfrac{10\pi^3 \sqrt{3}}{243} $$
    A demon  wind propelled me east of the sun
  • Franchement, quel bazar pour contourner la formule $\displaystyle \int_0^{\infty}\frac{x^{s-1}}{1+x}dx=\frac{\pi}{\sin \pi s}$ que propose sagement YvesM ci-dessus.
  • P. : Tu veux dire, quelle richesse de points de vue !
    La méthode que je propose plus haut.
    , permet sans doute de calculer* $\displaystyle \int_0^\infty \frac{\ln^{2n} x}{1+x^3}dx$ et très certainement on peut montrer que $\displaystyle \int_0^\infty \frac{\ln^{2n} x}{1+x^3}d=r\sqrt{3}\pi^{2n+1}$ avec $r$ un rationnel.

    (Un autre point de vue évoqué est d'utiliser la fonction beta d'Euler)

    *: il faut connaître toutefois la valeur de $\zeta(2n+2)$.

    PS:
    Je pense qu'on a, par exemple, :
    \begin{align}\int_0^\infty \frac{\ln^{4} x}{1+x^3}dx=\frac{34\sqrt{3}\pi^5}{729}\\
    \int_0^\infty \frac{\ln^{6} x}{1+x^3}dx=\frac{910\sqrt{3}\pi^7}{6561}\\
    \end{align}
    (cela semble fonctionner aussi pour $8,10$ mais les numérateurs,dénominateurs de $r$ (sous forme réduite) sont trop gros j'ai la flemme de les recopier )
  • Rien de tres specifique aux nombres pairs:
    $$\int_0^{\infty}\frac{(\log x)^k}{1+x^3}dx=3^{-k-1}\int_0^{\infty}\frac{(\log u)^ku^{2/3}}{1+u}du=
    3^{-k-1}\pi\left[\left(\frac{d}{ds}\right)^k\frac{1}{\sin (\pi s)}\right]_{s=1/3}.$$
  • P.: Mais la parité de ton $k$ a des conséquences sur le calcul que je propose plus haut.
    Pas sûr que la méthode fonctionne très bien pour calculer par exemple, $\displaystyle \int_0^\infty \dfrac{\ln^3 x}{1+x^3}dx$.
    (et plus généralement pour les $k$ impairs au moins égaux à $3$)

    Par ailleurs, ta formule me semble bizarre: pour $k$ pair il va y avoir un $\sqrt{3}$ en facteur. Je ne vois pas dans ta formule d'où va provenir ce $\sqrt{3}$.*

    Pour terminer, en réalité on a; $\displaystyle \int_0^\infty \dfrac{\ln^{2n+1} x}{1+x^3}dx=r\pi^{2n+2}$ avec $r$ un rationnel.

    *: je croyais que $\sin\left(\frac{\pi}{3}\right)$ était égal à $1/2$ :-D
  • En fait, on a:

    \begin{align}L&=\int_0^\infty\int_0^\infty \frac{\ln^n(xy)}{(1+x^3)(1+y^3)}dxdy\\
    &\overset{u=xy}=\int_0^\infty\int_0^\infty \frac{y^2\ln^n u}{(u^3+y^3)(1+y^3)}dxdy\\
    &=\int_0^\infty \left[\frac{\ln\left(\frac{1+y^3}{u^3+y^3}\right)}{3(u^3-1)}\right]_0^\infty \ln^n u\,du\\
    &=-\int_0^\infty \frac{\ln^{n+1} u}{1-u^3}du\\
    \end{align}

    Si on pose $\displaystyle J_k=\int_0^\infty \frac{\ln^k x}{1+x^3}dx$ on a, par ailleurs, $\displaystyle L=\sum_{k=0}^n \binom{n}{k}J_kJ_{n-k}$ ce qui donne, avec la valeur trouvée précédemment, une relation de récurrence qui permet de calculer les $J_k$ pour $k\geq 1$.

    On a par ailleurs, $\displaystyle \int_0^\infty \dfrac{\ln^n x}{1-x^3}dx=\int_0^1 \dfrac{\ln^n x}{1-x^3}dx+\underbrace{\int_1^\infty \frac{\ln^n x}{1-x^3}}_{y=\frac{1}{x}} dx=\int_0^1 \dfrac{\left(1+(-1)^{n+1}x\right)\ln^n x}{1-x^3}dx$

    On peut montrer, par intégration d'un développement en série entière que $\displaystyle \int_0^1 \dfrac{\ln^n x}{1-x^3}dx$ est un multiple rationnel de $A=\displaystyle \sum_{k=0}^\infty \dfrac{1}{(3k+1)^{n+1}}$ et $\displaystyle \int_0^1 \dfrac{x\ln^n x}{1-x^3}dx$ est un multiple rationnel de $B=\displaystyle \sum_{k=0}^\infty \dfrac{1}{(3k+2)^{n+1}}$
    Mais on connait la valeur de $C=\displaystyle \sum_{k=1}^{\infty} \dfrac{1}{(3k)^{n+1}}$ c'est un multiple rationnel de $\zeta(n+1)$. Cela explique pourquoi on va être capable de calculer aisément $\displaystyle \int_0^1 \dfrac{\left(1+(-1)^{n+1}x\right)\ln^n x}{1-x^3}dx$ quand $n$ est impair (on a $A+B+C=\zeta(n+1)$ )

    PS:
    J'ai trouvé cela cocasse quand le nombre de vues de ce fil était de $1644$ B-)
  • Bonsoir, je ne peux pas non plus résister à l'envie de donner ce calcul de $I = \displaystyle \int_0^{+\infty} \dfrac{\ln (x)}{1+x^n} \mathrm dx $:
    $$\
    displaystyle \int_0^{+\infty} \dfrac{\ln (x)^2}{1+x^n} \mathrm dx - \displaystyle \int_0^{+\infty} \dfrac{\ln (x+ 2\pi i)^2}{1+x^n} \mathrm dx \; = \; 2\pi i \sum_{a\in D} \bigl\{ \mathrm res_{z=a} \left( \dfrac{\ln (z)^2}{1+z^n}\right) \bigr\} .

    $$ Ceci se prouve en intégrant $\dfrac{\ln (z)^2}{1+z^n}$ sur un contour en forme de trou de serrure.
    On développe le carré :
    $$

    -4\pi i \int_0^{+\infty} \dfrac{\ln (x)}{1+x^n} \mathrm dx +4\pi^2 \int_0^{+\infty} \dfrac{1}{1+x^n} \mathrm dx \; = \; 2\pi i \sum_{a\in D} \bigl\{ \mathrm res_{z=a} \left( \dfrac{\ln (z)^2}{1+z^n}\right) \bigr\} .





    $$ On sépare partie réelle et imaginaire après avoir divisé par $-4\pi i$ :
    $$

    \int_0^{+\infty} \dfrac{\ln (x)}{1+x^n} \mathrm dx \; = \; -\frac{1}{2}\Re \Bigl\{ \sum_{a\in D} \mathrm res_{z=a} \left( \dfrac{\ln (z)^2}{1+z^n}\right) \Bigr\} .



    $$ Il reste à évaluer les résidus correspondant aux pôles $\rho _k = \zeta^k e^{i\pi /n}$ avec $\zeta = e^{2i\pi/n}$ pour $ 0 \leq k \leq n-1$ :
    Si $g(z) = \dfrac{1}{1+z^n}$, les résidus de $g$ sont : $



    \mathrm res_{z=\rho_k} g(z) = -\dfrac{\rho_k}{n},

    $
    donc $\mathrm res_{z=\rho_k} \left( \ln (z)^2 g(z)\right) = -\dfrac{\rho_k}{n} \log \left( \rho_k\right)^2 $
    Il faut prendre la détermination du logarithme correspondant à un argument compris entre 0 et $2\pi$ :
    $$
    \ln \left( \rho_k\right)^2 =- \left( \pi/n + 2k\pi/n \right)^2 \;= \; -(2k+1)^2\frac{\pi^2}{n^2} .

    $$ Finalement :
    $$
    \mathrm res_{z=\rho_k} g(z) = \rho_k\dfrac{\pi^2(2k+1)^2}{n^3}.


    $$ On a besoin de la partie réelle de la somme des résidus :
    $$
    \Re \bigl\{ \rho_k\bigr\} \; = \; \cos\left(\dfrac{(2k+1)\pi}{n} \right).

    $$ Soit :
    $$

    \int_0^{+\infty} \dfrac{\ln (x)}{1+x^n} \mathrm dx \; = \; -\frac{\pi^2}{n^3} \sum_{k=0}^{n-1} (2k+1)^2\cos\left(\dfrac{(2k+1)\pi}{n} \right).




    $$ On termine le calcul en évaluant la somme :
    $$
    C_{n,2} (x) \; = \; \displaystyle \sum_{k=0}^{n-1} (2k+1)^2\cos\left((2k+1)x \right) .

    $$ On reconnaît la dérivée seconde de :
    $$
    C_{n} (x) \; = \;\displaystyle \sum_{k=0}^{n-1} \cos\left((2k+1)x \right)


    $$ $C_n(x)$ se calcule classiquement en évaluant $\displaystyle \sum_{k=0}^{n-1} e^{i(2k+1)x} =e^{ix} \sum_{k=0}^{n-1} e^{i2kx} = e^{ix}\dfrac{e^{i2nx} - 1}{e^{i2x}-1} $

    Les formules d'Euler donnent :
    $$
    C_n(x) \; = \; \Re \Bigl\{ e^{inx}\dfrac{\sin(nx)}{\sin(x)} \Bigr\}.

    $$ Bilan : $$C_n(x) \; = \; \dfrac{\sin(2nx)}{2\sin(x)}.

    $$ Après avoir dérivé deux fois et pris la valeur de la dérivée seconde en $x=\frac{\pi}{n}$ :
    $$
    C_{n,2}\big(\frac{\pi}{n}\big) \; = \; 2n\dfrac{\cos ( \frac{\pi}{n})}{\sin^2(\frac{\pi}{n})}.



    $$ Finalement :
    $$
    \int_0^{+\infty} \dfrac{\ln (x)}{1+x^n} \mathrm dx \; = \;-\dfrac{\pi^2}{n^2} \; \dfrac{\cos ( \frac{\pi}{n})}{\sin^2(\frac{\pi}{n})}.

    $$ On peut alors remplacer l'entier $n$ par un complexe $s$ convenablement choisi par prolongement analytique.
    On peut vérifier que pour $n=3$, on obtient :
    $$
    \int_0^{+\infty} \dfrac{\ln (x)}{1+x^3} \mathrm dx \; = \; -\dfrac{2\pi^2}{27}.$$
    A demon  wind propelled me east of the sun
  • Comment on démontre $\displaystyle \int_0^{+\infty} \dfrac{\ln (x)}{1+x^n} \mathrm dx \; = \;-\dfrac{\pi^2}{n^2} \; \dfrac{\cos (\frac{\pi}{n})}{\sin^2(\frac{\pi}{n})}.$ sans le théorème des résidus ? :)
    Je suis donc je pense 
  • Par dérivation si tu supposes connu $\int_{0}^{+\infty}\frac{x^a}{1+x^n}\,dx$
    Le 😄 Farceur


  • Gebrane: tu veux dire $\displaystyle \int_0^{\infty} \dfrac{1}{x^s(1+x)}dx$ avec $0<s<1$
  • FDP On a pas déjà vu ce calcul je crois dans le fil d'Oshine ?
    $\int_0^{\infty}\frac{x^{a-1}}{1+x^b}dx=\frac{1}{b}\int_0^{\infty}\frac{u^{\frac{a-1}{b}-1+\frac{1}{b}}}{1+u}du=\frac{1}{b}\Gamma\left(\frac{a}{b}\right)\Gamma\left(1-\frac{a}{b}\right)=\frac{\pi}{b \sin \left(\frac{a\pi}{b}\right)}.$
    Le 😄 Farceur


  • @Quentino37: le théorème de dérivation d'une intégrale à paramètre est aussi un gros théorème d'analyse dont la preuve n'est pas très sympathique et dont les hypothèses devraient être vérifiées en fonction de la situation étudiée.

    Dans le même genre, si tu considères $F(s) =\int_0^{+\infty} \dfrac{1}{1+x^s} \mathrm dx $ pour $\Re (s) > 1$, en dérivant sous le signe d'intégration, tu obtiens $F'(s) =-\int_0^{+\infty} \dfrac{x^s \ln(x)}{(1+x^s)^2} \mathrm dx $ et en écrivant $\dfrac{x^s }{(1+x^s)^2} = \dfrac{1+x^s }{(1+x^s)^2} - \dfrac{1}{(1+x^s)^2} = \dfrac{1}{1+x^s} - \dfrac{1}{(1+x^s)^2}$, on arrive à:

    $$F'(s) =- \int_0^{+\infty} \dfrac{ \ln(x)}{1+x^s} \mathrm dx + \int_0^{+\infty} \dfrac{\ln(x)}{(1+x^s)^2} \mathrm dx $$
    Edit: pour un $ de plus...
    A demon  wind propelled me east of the sun
  • Gebrane: "ma" formule avait l'avantage de n'avoir qu'un seul paramètre et est une formule standard.

    $\displaystyle \int_0^\infty \dfrac{\ln x}{1+x^n}dx\overset{y=x^n}=\frac{1}{n^2}\int_0^\infty \dfrac{y^{\frac{1}{n}-1}\ln y}{1+y}dy=\frac{1}{n^2}\frac{\partial}{\partial s}\left.\int_0^\infty \frac{y^{\frac{1}{n}-1}y^s}{1+y}dy\right |_{s=0}$
  • Problème 12260 (AMM):

    Calculer $\displaystyle \int_0^\infty \dfrac{\sin^2 x-x\sin x}{x^3}dx$(*)

    NB: On peut calculer de façon élémentaire cette intégrale: c'est à dire sans connaître la valeur de $\displaystyle \int_0^\infty \dfrac{\sin x}{x}dx$ et sans gros théorème "marteau-pilon".**

    *: la valeur était donnée dans l'énoncé original.

    **: Apparemment, il existerait un théorème qui permet de calculer $\displaystyle \int_0^\infty \dfrac{f(x)\sin x}{x}dx$ quand $f$ a de bonnes propriétés. On n'en a nul besoin ici.
  • Il suffit de faire deux IPP pour etre ramene a deux integrales birn connues egales respectivement a 1/2 et -Ln(2)
    fjaclot
  • Gebrane: Ce que tu dis reviens à utiliser une méthode de calcul analogue à celle qu'on peut utiliser pour calculer $\displaystyle \int_0^\infty \dfrac{\sin x}{x}dx. Pas vraiment élémentaire.

    Comme déjà indiqué un raisonnement accessible à un étudiant de première année de licence permet de faire ce calcul.
    (pas d'intégrale double, pas de dérivation sous le signe intégral)

    NB: TL, j'imagine, signifie transformée de Laplace.
  • FJaclot: J'avais commencé comme toi, mais ne voyant pas de conclusion simple et ayant observé un fait remarquable sur l'intégrale initiale je suis passé à autre chose qui s'est avéré concluant.
    Comme déjà indiqué on n'a besoin d'aucun outil élaboré du calcul intégral.
  • FDP à force de se restreindre, on peut devenir borné

    Oui les TL , c'est les transformées de Laplace , j'utilise cette identité

    https://wikimedia.org/api/rest_v1/media/math/render/svg/6f622261e2084edd910d0fa39e34e454165c4734

    https://en.wikipedia.org/wiki/Laplace_transform#Evaluating_integrals_over_the_positive_real_axis
    Le 😄 Farceur


  • Gebrane: Probablement un certain nombre de gens vont faire ce calcul comme tu le ferais et c'est peut-être trop facile ainsi.
    La formule "marteau-piqueur" à laquelle je pensais ne s'applique pas ici semble-t-il.
  • La methode que tu cites demande que ton f soit \pi périodique et ce n'est pas le cas
    Le 😄 Farceur


  • Gebrane: Oui, en effet mais de toute façon ce n'est pas ce que j'ai utilisé*. Je croyais initialement que le théorème cité allait "écraser" le problème mais heureusement il semble que ce ne soit pas le cas.

    *: je n'aurais pas été chouiner sur l'utilisation d'une transformation intégrale de Laplace si on avait pu utiliser le théorème cité et que j'avais utilisé ladite formule.
  • On intègre sur l'intervalle $[0;\infty[$, l'intégrande est le produit de $1/x$ par une expression et le résultat comporte la valeur $\ln 2$ on a envie de pouvoir mobiliser...la formule de Cauchy*-Frullani.
    Peut-être une fausse piste, je ne vois pas à cet instant comment exploiter cette idée.

    *: autrement le diable va sortir de sa boîte. B-)-
  • Pourquoi on se fatigue, wolphi connait une primitive https://www.wolframalpha.com/input/?i=\int+(sin^2+x-xsin+x)+/+x^3

    Elle n'est pas trop difficile à établir seulement des ipp après linéarisation de $sin^2 x$
    Le 😄 Farceur


  • Gebrane: le calcul des limites aux bornes pourrait s'avérer coton. On peut faire beaucoup plus simple.
  • Tu n'aime pas les TL, tu n'aimes pas les ipp de FJaclot, tu n'aimes pas utiliser une primitive gratuite, je ne sais pas ce que tu cherches
    > LA GUERRE :-D
    Le 😄 Farceur


  • FDP Eureka j'ai trouvé une belle façon incroyable comment utiliser Frullani

    1- Une ipp
    2- séparer intelligemment l’intégrale en deux
    3- pour l'une utiliser Frullani, pour l'autre une primitive simple saute aux yeux
    4- le terme par Frullani donne -ln(2), l'autre par primitivation donne 1/2

    Je te laisse la trouver par toi même. Youppi je suis content de moi :-D
    Le 😄 Farceur


  • Gebrane: Bravo ! (il reste à voir le calcul). Cela ressemble à mon calcul, Frullani en moins. B-)-
  • Cauchy-Frullani, SVP.
  • Gebrane a écrit:
    Tu n'aime pas les TL, tu n'aimes pas les ipp de FJaclot

    Si cela débouche sur un résultat cela me convient mais comme déjà indiqué j'ai commencé à faire de même, le $1/x^3$ nous y invite et il n'y a pas de problème de convergence si je me souviens bien. Après, je suis curieux de voir la fin du calcul quand tu as une intégrale sous la forme $\displaystyle \int_0^\infty \dfrac{f(x)}{x}dx*$, il faut en faire sortir un logarithme tout de même. B-)-

    *: et je serai étonné d'apprendre qu'on puisse continuer par IPP.

    PS: le diable est sorti de sa boîte. :-D
  • Ok FDP , je te montre mon calcul

    1-$\displaystyle \int_0^\infty \dfrac{\sin^2 x-x\sin x}{x^3}dx=\frac 12 \displaystyle \int_0^\infty \dfrac{\sin (2x)-\sin x -x\cos x}{x^2}dx$
    2- On separe en deux
    $\frac 12 \displaystyle \int_0^\infty \dfrac{\sin (2x)-2\sin x +\sin x-x\cos x}{x^2}dx \\
    =\frac 12 \displaystyle \int_0^\infty \dfrac{\sin (2x)-2\sin x }{x^2}dx +\frac 12 \displaystyle \int_0^\infty \dfrac{\sin x-x\cos x}{x^2}dx \\
    = \frac 12 \displaystyle \int_0^\infty \dfrac{ \dfrac{2\sin (2x) }{2x} -\dfrac{2\sin (x) }{x}}{x} dx -\frac 12 \displaystyle \int_0^\infty (\dfrac{\sin x}{x})'dx \\
    =\frac 12 \displaystyle \int_0^\infty \dfrac{f(2x)- f(x) }{x} dx -\frac 12 \displaystyle \big[\dfrac{\sin x}{x})\big ]_0^{+\infty}\\
    =-\ln 2+\frac 12 $

    C'est ma grande performance de l'année, svp des likes :-D
    Le 😄 Farceur


  • \begin{align}J&=\int_0^\infty \frac{\sin^2 x-x\sin x}{x^3}dx\\
    J(\epsilon)&^\overset{\epsilon>0}=\int_\epsilon^\infty \frac{\sin^2 x-x\sin x}{x^3}dx\\
    &=\int_\epsilon^\infty \frac{\sin^2 x}{x^3}dx-\int_\epsilon^\infty \frac{\sin x}{x^2}dx\\
    &\overset{\text{IPP}}=-\frac{1}{2}\left[\frac{\sin^2 x}{x^2}\right]_{\epsilon}^\infty+\frac{1}{2}\underbrace{\int_{\epsilon}^\infty\frac{\sin(2x)}{x^2}dx}_{y=2x}-\int_\epsilon^\infty \frac{\sin x}{x^2}dx\\
    &=\frac{1}{2}\frac{\sin^2 \epsilon}{\epsilon^2}+\int_{2\epsilon}^\infty \frac{\sin x}{x^2}dx-\int_\epsilon^\infty \frac{\sin x}{x^2}dx\\
    &=\frac{1}{2}\frac{\sin^2 \epsilon}{\epsilon^2}-\int_\epsilon^{2\epsilon} \frac{\sin x}{x^2}dx\\
    &=\frac{1}{2}\frac{\sin^2 \epsilon}{\epsilon^2}-\int_\epsilon^{2\epsilon} \frac{\sin x-x}{x^2}dx-\int_\epsilon^{2\epsilon}\frac{1}{x}dx\\
    &=\frac{1}{2}\frac{\sin^2 \epsilon}{\epsilon^2}-\int_\epsilon^{2\epsilon} \frac{\sin x-x}{x^2}dx-\ln 2
    \end{align}
    La fonction $f:x\rightarrow \dfrac{\sin x-x}{x^2}$ définie sur $[0;1]$, avec $f(0)=0$, est continue.

    Pour $\displaystyle 0<\epsilon<\dfrac{1}{2},\left|\int_\epsilon^{2\epsilon} \frac{\sin x-x}{x^2}dx\right|\leq \epsilon \times\sup_{x\in [0;1]} \left|f(x)\right|$.

    ($f$ est bornée sur $[0;1]$ puisque $f$ est continue sur $[0;1]$)

    Donc, $\displaystyle \lim_{\epsilon \rightarrow 0}\int_\epsilon^{2\epsilon} \dfrac{\sin x-x}{x^2}dx=0$.

    En outre, $\displaystyle \lim_{\epsilon \rightarrow 0}\dfrac{\sin \epsilon}{\epsilon}=1$.

    Donc,

    \begin{align}J&=\lim_{\epsilon \rightarrow 0}J(\epsilon)=\boxed{\frac{1}{2}-\ln 2}\end{align}
  • Bravo FDP, un calcul bien soigné
    Le 😄 Farceur


  • Gebrane: Ta solution est bien, mais elle est moins élémentaire que la mienne ;)

    Je me demande si on ne peut pas faire plus simple dans l'utilisation de la formule de Frullani maintenant que tu as donné une provenance probable du $1/2$ de la réponse.
  • Tu es incorrigible FDP, tu t'es demandé comment utiliser Cauchy-Frullani et maintenant tu dis que ma méthode n'est pas élémentaire. Moi j'ai répondu seulement à ta question mais ma méthode préférée est celle des TL
    Un bonus si tu veux je peux t’expliquer la suite de la méthode de fjaclot
    Le 😄 Farceur


  • Gebrane: En m'inspirant de ton calcul on a que:

    $\displaystyle \int_0^\infty \dfrac{\sin^2 x -\frac{3}{2}x\sin x+\frac{1}{2}x^2\cos x}{x^3}=-\ln 2$

    Il ne reste plus qu'à frouver une fonction $F$ et des réels $a,b$ tels que $\displaystyle \dfrac{\sin^2 x -\frac{3}{2}x\sin x+\frac{1}{2}x^2\cos x}{x^3}=\dfrac{F(ax)-F(bx)}{x}$
  • Gebrane: Ta solution est bien mais derrière la formule de Cauchy*-Frullani il y a certainement de l'intégrale double/de la dérivation sous le signe intégrale**. :-D

    *: pour que le diable retourne dans sa boîte.
    **: pas sûr, ça se trouve une manipulation comme celle que j'ai utilisée fait le travail.

    Pour la suite du calcul de FJaclot tu as toute mon attention. B-)-

    PS:

    J'étais parvenu à obtenir une expression du type $\displaystyle \int_0^\infty \dfrac{F(x)}{x}dx$ mais je n'ai pas vu qu'on pouvait "sortir" un bout du numérateur comme tu l'as fait alors j'ai senti que je m'engageais sur une voie sinueuse d'autant plus que je n'avais aucune idée de comment faire apparaître un logarithme (J'ai pensé à Frullani bien plus tard) et c'est à ce moment que je me suis dit que j'avais intérêt à couper l'intégrande initiale en deux (je voyais bien qu'un bout de l'intégrale était semblable à l'autre bout à intégration par parties près) mais qu'à cause du problème de convergence en $0$ il fallait travailler sur des limites d'intégrales.
  • Calcul imaginé par gebrane de FJaclot

    en édition voir message suivant
    Le 😄 Farceur


  • Et on se retrouve à calculer la limite de $\displaystyle \lim_{x\rightarrow 0}\int_{x}^{2x} \dfrac{\cos t}{t}dt$

    Et on peut utiliser un truc analogue à ce que j'ai fait plus haut:

    Pour $\displaystyle x>0, \int_{x}^{2x} \dfrac{\cos t}{t}dt=\int_{x}^{2x} \dfrac{1}{t}dt-\int_{x}^{2x} \dfrac{1-\cos t}{t}dt=\ln 2-\int_{x}^{2x} \dfrac{1-\cos t}{t}dt$

    La dernière intégrale tend vers $0$ quand $x$ tend vers $0$ l'intégrande peut être prolongée continûment en $0$.
    (l'intégrande est donc bornée sur $[0;1]$)

    PS:
    On peut aussi utiliser la formule de Cauchy-Frullani mais ce serait compliquer le travail à mon humble avis.
  • FDP
    Pour la première ipp pas de problème, mais attention pour la deuxième ipp le problème est en $+ \infty.$
    J'ai réussi à contourner le problème et je trouve que notre intégrale est tout simplement
    $$
    \frac 12+\int_0^{+\infty } \frac{\cos(2 x) -\cos(x) }x .

    $$ Es-tu d'accord ?
    Explications

    La première ipp passe sans problème
    $\displaystyle \int_0^\infty \dfrac{\sin^2 x-x\sin x}{x^3}dx=\frac 12 \displaystyle \int_0^\infty \dfrac{\sin (2x)-\sin x -x\cos x}{x^2}dx$
    La deuxième pose un problème en $+\infty$, on considère donc $F(X)= \displaystyle \int_0^X \dfrac{\sin (2x)-\sin x -x\cos x}{x^2}dx$. Par ipp
    $F(X)= -\big [\dfrac{\sin (2x)-\sin x -x\cos x}{x}\big ]_0^X+ \int_0^X \dfrac{2\cos (2x)-2\cos x +x\sin x}{x}dx\\
    = -\dfrac{\sin (2X)-\sin X -X\cos X}{X}+ \int_0^X \dfrac{2\cos (2x)-2\cos x}{x}dx
    -(\cos X-1)\\
    =-\dfrac{\sin (2X)-\sin X }{X}+ \int_0^X \dfrac{2\cos (2x)-2\cos x}{x}dx +1 \to^{X\to +\infty}\quad 2\int_0^{+\infty} \dfrac{\cos (2x)-\cos x}{x}dx +1 $






    Ajout. J'ouvre un fil sur le calcul de cette intégrale, je crois c'est une richesse de méthodes.
    Le 😄 Farceur


  • Tu vois la méthode de flajco etait plus simple avec ces deux ipp
    Le 😄 Farceur


Connectez-vous ou Inscrivez-vous pour répondre.